Search found 22 matches


Hi taryn123, For the quant portion, I stuck with studying Manhattan gmat and I made sure that I understood each problem in the OG. I did not look into any books. I felt understanding each problem is more important that doing many problems. That helped me tremendously. For some reason I keep scoring ...

by batman73

Mon Aug 17, 2009 7:16 am
Forum: I just Beat The GMAT!
Topic: from 490 to 590 to 610...and not stopping
Replies: 3
Views: 2225

from 490 to 590 to 610...and not stopping

Hi, I took the GMAT on Saturday. I have been studying religiously and I was able to raise my quant score from 36 to 47. On the other hand my verbal score went from 36 to 27. I am not sure what went wrong, but something did. Below is a breakdown of my score. TEST / DATE/SCORE/Q/V TEST DAY 3/19/09 590...

by batman73

Tue Aug 11, 2009 7:23 am
Forum: I just Beat The GMAT!
Topic: from 490 to 590 to 610...and not stopping
Replies: 3
Views: 2225

I have the same problem too. It is not clicking for me as well. If anyone has any suggestions, I would greatly appreciate it

by batman73

Fri Aug 07, 2009 8:13 am
Forum: Data Sufficiency
Topic: Absolute value and Inequalities
Replies: 2
Views: 1279

The word problem gives us the ratio: 1C:4S so we can write this as: x+4x=Total If we are given the total amount of cushions, then we can solve for the equation (1) Last week the factory produced more chairs than sofas. This gives us no useful information (2) Last week the factory produced a total of...

by batman73

Wed Aug 05, 2009 6:54 am
Forum: Data Sufficiency
Topic: A small factory that produces only upholstered chairs and so
Replies: 4
Views: 1833

GCF and LCM problem from Manhattan Gmat

Is the integer divisible by 6?

1)The GCF of z and 12 is 3.
2)THE GCF of z and 15 is 15.

Can someone help me with this problem? A


when the GCF of z and 12 is 3...it is possible that z is the number 3, which woul make it insufficient.

by batman73

Tue Aug 04, 2009 6:31 pm
Forum: Data Sufficiency
Topic: GCF and LCM problem from Manhattan Gmat
Replies: 2
Views: 2360

"Critique" MUST modify the book since it is directly after the comma. You must modify the noun rather than the critiquing style. Also Parallelism must be considered.

Therefore, A is the correct answer

Hope that makes sense

by batman73

Tue Aug 04, 2009 6:55 am
Forum: Sentence Correction
Topic: OG 53
Replies: 3
Views: 1381

Re: GMATprep2: Is x+y<1?

nhai2003 wrote:Is x+y<1?

1) x<8/9
2)y<1/9

OA later.
Thanks guys

it cannot be a or b because you need the x and y to solve the problem

Combining both statements: get same denominator

(64/72) + (9/72)< 1

73/72< 1

So x+y can be greater than 1 or less than 1.....ANSWER IS E.

by batman73

Tue Jul 14, 2009 8:37 pm
Forum: Data Sufficiency
Topic: GMATprep2: Is x+y<1?
Replies: 6
Views: 3152

Re: which of the following cannot be the median

which of the following cannot be the median of the three ordered positive integers x,y and z? x z x+z x+z/2 x+z/3 I got the correct answer (c) but was thrown off-guard by the word "ordered".What does it mean?At first glance I thought it mens that they are ordered in ascending/descending b...

by batman73

Tue Jul 14, 2009 7:38 pm
Forum: Problem Solving
Topic: which of the following cannot be the median
Replies: 3
Views: 3386

Re: OG q39, page 280

1. A circular tub has a band painted around its circumference, as shown below. What is the surface area of this painted band? x height) (1) x = 0.5 (2) The tub is 1 meter high. Guys please help me with this one Thanks The tube has the surface area of SA= 3.14(r)^2 + 2(3.14)rh. In order to solve thi...

by batman73

Mon Jul 13, 2009 8:30 pm
Forum: Data Sufficiency
Topic: OG q39, page 280
Replies: 1
Views: 1338

gmatprep k does not = -1,0,1, is (1/k) > 0?

gmatprep k does not = -1,0,1, is (1/k) > 0?

1. 1/(k-1)>0

2. 1/(k+1)>0


The answer is A. I am getting D. Can someone please help with this problem. Thanks in advance.

by batman73

Mon Jul 13, 2009 1:54 pm
Forum: Data Sufficiency
Topic: gmatprep k does not = -1,0,1, is (1/k) > 0?
Replies: 6
Views: 1532

Because this is an inference problem, I believe the best answer is E.

by batman73

Mon Jun 08, 2009 7:59 am
Forum: Reading Comprehension
Topic: 1000 rc 4
Replies: 5
Views: 2741

"Look at those cars. Theirs is really ugly."

"Look at those cars. Theirs is really ugly."

I believe this sentence discusses several cars in the first sentence, but in the second it is possessive to a group. Categorizing in a group makes it singular. That is the best analysis I can give. I believe the sentence is correct

by batman73

Mon Jun 08, 2009 7:31 am
Forum: GMAT Verbal & Essays
Topic: "Look at those cars. Theirs (is/are) really ugly"
Replies: 1
Views: 2386

You have a great Quant score. Focus more on the verbal next time around. If everyone gave up this forum would not exist. I would suggest you just spend more time on the Verbal portion. You might have been been exhausted by the latter part of the test. Take a break, 1 or 2 weeks, and get the ball rol...

by batman73

Thu Apr 02, 2009 8:23 am
Forum: I just Beat The GMAT!
Topic: 550 (48/19) I am giving up.
Replies: 14
Views: 3850

hi

I think any higher becomes irrelevant. Be happy with your score and start applying. Congrats. Best of luck on the application process

by batman73

Fri Mar 27, 2009 8:20 am
Forum: I just Beat The GMAT!
Topic: 720 - did i beat the gmat or did it beat me?
Replies: 11
Views: 3537

This is the first step

Hi,
I would definitely prepare to take the test again. I studied for 2 months, took the gmat and got a 490. I just retook it and got a 590 (43Q 29V). Do not give up..have the focus and the drive and I am sure you will break the 650 mark. Good luck on your studying.

by batman73

Fri Mar 27, 2009 7:31 am
Forum: I just Beat The GMAT!
Topic: beaten by GMAT, 510 (Q31, V29)
Replies: 5
Views: 3245